Tyrone works in a grocery store and he puts boxes on shelves. There are 4 shelves in the cereal section of the store. All shelves hold the same number of boxes of cereal. If there are 80 boxes of cereal on the shelves, how many boxes are on each shelf? In complete sentence

Answers

Answer 1

Answer:

Each shelf can hold 20 boxes of cereal.

Step-by-step explanation:

Start with 80 boxes of cereal: (80)

There are 4 shelves for each box of cereal: (x/4)

Thus, we get the expression: 80/4=

(You may want to simplify this; giving you the answer of 20.)


Related Questions

As Earth circulates the sun, it travels at a
speed of 20 miles per second. Convert
this speed to kilometers per second. At
this speed, how many kilometers will
Earth travel in 8 seconds? Assume that 1
mile is equal to 1.6 kilometers.

Answers

If 1 mile is equal to 1.6 kilometres, then 20 miles will be equal to 20 x 1.6 = 32 kilometres

So it travels 32 kilometres per second


For the second part, if the earth travels 32 kilometres in 1 second, then it will travel 32 x 8 kilometres in 8 seconds, which is equal to 256 km

Identify the common difference of the sequence: –8, –1, 6, 13,

Answers

They are skipping -7, 5, -7.

The common difference of the sequence is 18.

factor this polynomial completely x^2-10x+25

Answers

x^2-10x+25

=x^2-5x-5x+25

x(x-5)-5(x-5)

=(x-5)(x-5)

x=5 , x=5

hope it's helpful to you

[tex]x^2 -10x +25\\\\=x^2 -2 \cdot 5 \cdot x +5^2\\\\=(x-5)^2\\\\=(x-5)(x-5)[/tex]

Lamar wants to replace a glass window in his restaurant. The window is in the shape of a square. Its side lengths are 11 feet. Suppose glass costs $5 for each
square foot. How much will the glass cost to replace the window?
sa
x 5 ?

Answers

Answer:

55

55x11 = 55

(Meeting character limit)

Answer:

I believe it would be 605$

6. The California Tiger Salamander is an endangered species, which decreases at the rate of 4.6% per year in a habitat that currently has 60 of them. Write an exponential function and find how many California Tiger Salamanders will be left after 4 years.

7. Factor and solve the following equation 2x2 + x - 21 = 0.

8. Alvin throws the football to a receiver who jumps up to catch the ball. The height of the ball over time can be represented by the quadratic equation -4.9t2 + 7.5t + 1.8 = 2.1 . This equation is based on the acceleration of gravity -4.9 m/s2, the velocity of his pass is 7.5 m/s and releases the football at a height of 1.8 meters, and the height where the receiver catches the ball of 2.1 meters. Put the equation in standard form and then solve by using the quadratic equation.

9. Examine the graph of f(x) and the table that contains values of g(x). Which function has a greater rate of change over the interval [0,2]? Explain your answer.

Answers

[tex]\rule{2000pt}{1000000pt}[/tex]

a*3a^(2) help quickly

Answers

the answer is 6a^3 :))

Determine if a relation is a function {(-1,5), (2, 7), (-3,2), (-1, 1)}

Answers

The relation is NOT a function

Answer:

Step-by-step explanation:

A relation is only a function if ALL the x-values (the first number in the pairs) ONLY have one partner.

Check that -1

At first that -1 is paired up with a 5 in the ordered pair (-1, 5) but then later... that -1 is down there hanging out with a 1 in the last ordered pair (-1, 1).

This means that this relation (all the pairs lumped together) is NOT a function. Just because of that -1 pairing up with more that one partner.

Madelyn hikes 190.5 meters up a
mountain. If she is 62% of the way
up, how tall is the mountain? Round
to the nearest whole meter.

Answers

It’s either C or D, but i’m pretty sure it’s C

3/2x-5=-4/7x+1/5 please ​

Answers

The value of x in the equation is 364/145

The equation is given as:

[tex]\frac 32x - 5= -\frac 47x + \frac 15[/tex]

Collect the like terms

[tex]\frac 32x + \frac 47x= 5 + \frac 15[/tex]

Take the LCM

[tex]\frac{21x + 8x}{14} = \frac{25 + 1}{5}[/tex]

Evaluate the sum

[tex]\frac{29x}{14} = \frac{26}{5}[/tex]

Cross multiply

[tex]145x = 364[/tex]

Divide both sides by 145

[tex]x = \frac{364}{145}[/tex]

Hence, the value of x in the equation is 364/145

Read more about equations at:

https://brainly.com/question/14323743

Cynthia gave 15 of her toys to her little sister. If she had 25 toys, how many did she give away?

Answers

5

she gave her sister 5 toys.

Correction to the question - It is not 15 it 1/5th.

According to the given question Cynthia had 25 toys and she gave 1/5th of her toys to her little sister.

So, The total number of toys is 25 to find 1/5th of it we need to multiply the total number of toys she had and the fraction she gave.

(25 x 1/5) = 5. so the number of toys she gave away to her little sister is 5.

To solve more of these kind of arithmatic and alzebraic word problems just search on web for word problems on arithmatic and  alzebra.

Learn more about word problems here :

https://brainly.com/question/2610134

#SPJ2

y=5/9x+9, find the slope

Answers

the slope would be 4/9

I need help I need help

Answers

Answer:

click on image

Step-by-step explanation:

click on the image

What is the slope of the line

Answers

Answer:

-1.5

Step-by-step explanation:

Find the equation of a line which contains the point (2, 5) and is parallel to the line y = 3x + 5.

Answers

Answer:

The equation of a line which contains the point (2, 5) and is parallel to the line y = 3x + 5 is

y = 3x - 1

A line is parallel to another when they both have the same slope.

The two equations above have a common slope of 3.

I hope this helped. Have a good new year!

I SWEAR IF I GET ANOTHER LINK I WILL SCREAM Parth created a Box-and-Whisker Plot to show the average weight of the fish he caught over the weekend. What information from the 1st Quartile does the graph give you?




A. Three fourths of the fish he caught were larger than 22 pounds.

B. The average weight of the fish he caught was 40 pounds.

C. Three fourths of the fish he caught were less than 22 pounds.

D. The largest fish he caught was 63 pounds.

Answers

Answer:

C. Three fourths of the fish he caught were less than 22 pounds.

Step-by-step explanation:

Hope it helps! At least you didn't scream Imao

Answer:

C

Step-by-step explanation:

i answered so the other person gets the brainliest they deserve!

Which answer choice shows the expression below factored correctly?
9x–15

Answers

whats the answer choices??

Answer:

[tex]3(3x-5)[/tex]

Step-by-step explanation:

First, you have to rewrite 9 as 3 * 3 and 15 as 3 * 5

Finally, you have to factor out the common term 3 to get [tex]3(3x-5)[/tex]

Hope this helps!

P.S next time list the answer choices it would help alot :)

electrons execute 1.4*10^6 revolutions per sec in a magnetic field of flux density
can you give explanation too​

Answers

The charge of the electrons is [tex]8.02 \times 10^{-19} \ C[/tex].

The given parameters:

Number of revolution of the electron, N = 1.4 x 10⁶ rev per secFlux density, B = 10⁻⁵ TMass of electron, m = 9.11 x 10⁻³¹ kg

The angular speed of the electron is calculated as follows;

[tex]\omega = 2\pi N\\\\\omega = 2\pi \ \frac{rad}{1 \ rev} \times 1.4 \times 10^6 \ rev/s\\\\\omega = 8.798 \times 10^6 \ rad/s[/tex]

The charge of the electrons is calculated as follows;

[tex]F_c= m\omega ^2 r\\\\F_{B}= qvB = q\omega r B \\\\F_c = F_B\\\\m\omega ^2 r = q\omega r B \\\\q = \frac{m\omega ^2 r}{\omega r B} \\\\q = \frac{m \omega }{B} \\\\q = \frac{9.11 \times 10^{-31} \times 8.798 \times 10^6}{10^{-5}} \\\\q = 8.02 \times 10^{-19} \ C[/tex]

The charge of the electrons is [tex]8.02 \times 10^{-19} \ C[/tex].

The complete question here:

Electrons execute 1.4*10^6 revolutions per sec in a magnetic field of flux density 10⁻⁵ T. What will be the value of charge of the electron?

Learn more about magnetic force of electrons here: https://brainly.com/question/3990732

12 cars in 15 hours how many cars is in 10 hours

Answers

Answer:

8

Step-by-step explanation:

12 cars in 15 hours how many cars is in 10 hours

12 : 15 = x : 10

x = 12 * 10 : 15

x = 8

-------------------

check

12 : 15 = 8 : 10

0.8 = 0.8

the answer is good

Simplify the expression by combining like terms.
6x + 9 + 2x + 4

Does anyone know the answer?

Answers

Answer:

8x + 13

Step-by-step explanation:

"Like terms" means the same kind of algebraic expression...the 6x and the 2x are "like terms" because they both have an x in them. The 9 and the 4 are both constants, which means they are plain numbers. So 6x and 2x is 8x .

9+4 is 13 just like elementary school.

So all together,

6x + 9 + 2x + 4

6x + 2x + 9 + 4

8x + 13

We can rearrange the terms because the problem has all addition in between the terms.

PLEASE HELP ILL GIVE BRAINLIESTTTT

Answers

Answer:

27

Step-by-step explanation:

Because OC biscts AOB, then COA=COB

Therefore AOB is twice COB

10x+4 = 2(6x-3)

10x+4 = 12x -6

2x = 10

x = 5

Therefore COA is 5x6-3 = 27

Answer:

9 degrees

Step-by-step explanation:

OK so we know that m<AOB is equal to the equation 10x+4. With this in mind, we also know that m<COB equals 6x-3. The ray OC intersects m<AOB, so this means that m<COB plus m<COA equals m<AOB.

First, we find m<COA by reversing the equation around to solve for m<COA.

m<AOB - m<COB = m<COA

Then, we Pug in the angles and solve.

m<AOB = 10x+4

m<COB = 6x-3

10x+4 - 6x-3 = 4x+1

Here is where things get interesting. See those 2 lines in each angle? That means they are equal in measure.

So what you do is simple, find what x equals in order to make this equation below true.

6x-3 = 4x+1

The answer for x is 2. So then just plug in to solve for m<COA and you're done!!

4(2)+1 =9

lil bonus: you can also get the same angle degree by plugging  2 into m<COB

A car traveling at 48 mph overtakes a cyclist who is riding at 16mph had a 3 hour head start how far from the starting point does the car overtake the cyclist?

Answers

The car overtakes the cyclist at a distance of 72 miles from the starting point.

,Speed is the ratio of distance travelled to time taken. It is given by:

Speed = distance / time

Let the car overtake the cyclist at distance d and time t.

For the car:

48 = d / t

d = 48t

For the cyclist:

16 = d / (t + 3)

d = 16t + 48

48t = 16t + 48

32t = 48

t = 1.5 hours

d = 48(1.5) = 72 miles

The car overtakes the cyclist at a distance of 72 miles from the starting point.

Find out more on speed at: https://brainly.com/question/3004254

If a triangle has a base of 6' and a height of 4' its third side would be what?

Answers

Answer:

≈7.2

Step-by-step explanation:

hope this helps

Answer: ≈7.2

Explanation: a^2+b^2=c^2
6^2+4^2=c^2
36+16=c^2
52=c^2
c= 7.211102550927979…
c≈7.2

The given number line shows the solutions to which inequality

Answers

Answer:

B (x-7<-6)

Step-by-step explanation:

x < 1

I thought of a number. My friend thought of a number too. A third of my number is a half of his number. Who's number is bigger?

Answers

Answer:

yoours

Step-by-step explanation:

thanks again for your time and consideration and I am a kid I am a kid

Find the value of x so that the line passing through (-3,7) and(x,5) has a slope of -4

Answers

Answer:

x= -5/2 or -5.2

Step-by-step explanation:

you know the equation: y=mx+b, so we can substitute the -3 for x, the 7 for y and the -4 for m:

7=-4(-3)+b

now solve:

7=12+b

subtract 12 on both sides:

b=-5

now what you want to do is substitute it in and solve for x:

y=-4x-5

so:

5=-4x-5,

solve:

10=-4x

x=-5/2!!!!!!!!!!!!!!

hope this helped :)

Simplify 5(2x−1)+(−4x+8) . Write your answer in factored form.

please make it simple :)

Answers

Answer:

6x+3

Step-by-step explanation:

I hope this is what you where looking for

aproxima a las unidades de mil y obtén los resultados estimados

Answers

Answer:

hi

Step-by-step explanation:

I NEED MORE HELP PLZZ

Answers

Answer:

  1A: all real numbers

  1B: x = {-3, 0, +18}

  1C: (-3, 0) U [3, 18)

  2A: h(x) = -1/160x^2 +1/2x

  2B: focus: (40, -30); directrix: y = 50; axis of symmetry: x = 40

Step-by-step explanation:

1A:

The graph is attached. The domain is all real numbers.

__

1B:

The x-intercepts for x < 3 are the zeros of the quadratic:

  x^3 -9x = 0

  x(x^2 -9) = 0

  x(x +3)(x -3) = 0   ⇒   x = 0, -3, +3 . . . . x=3 is not in the domain of this piece

The x-intercepts for x ≥ 3 are the zeros of the log function:

  -log4(x -2) +2 = 0

  log4(x -2) = 2 . . . . . . add log4(x -2)

  x -2 = 4^2 . . . . . . . . . take the anti-log

  x = 2 +16 = 18 . . . . . . add 2

The x-intercepts of g(x) are x = {-3, 0, +18}.

__

1C:

The graph shows the function is positive on the intervals ...

  (-3, 0) U [3, 18)

__

2A:

The vertex of the curve is given as (40, 10). The vertical scale factor will make the function 0 at x=0.

  h(x) = a(x -40)^2 +10

  h(0) = 0 = a(-40)^2 +10 = 1600a +10

  a = -10/1600 = -1/160 = -1/(4·40)

The equation of the parabola is ...

  y = -1/(160)(x -40)^2 +10

In standard form, this is ...

  y = -1/160x^2 +1/2x

__

2B:

When we computed the leading coefficient (a) in Part A, we found it to be -1/(4·40). This corresponds to 1/(4p) where p=-40 is the focus to vertex distance. The negative sign means the focus is 40 units below the vertex, at (40, -30). The corresponding location of the directrix is 40 units above the vertex, at y=50.

Of course the axis of symmetry is the vertical line through the vertex: x = 40. These are shown in the second attachment.

Solve the Inequality for x.
-5x + 20 < 5

A) x > 5

B) x < 3

C) x > 3

D) x < -5

Thanks!!! :D

Answers

Answer:

x>3

Step-by-step explanation:

-5x+20<5

-5x<5-20

-5x<-15

5x>15

x>15/5

x>3

Answer:

A) x > 3

Step-by-step explanation:

[tex]-5x + 20 < 5\\\rule{150}{0.5}\\-5x + 20 - 20 < 5 - 20 \\\\-5x < -15 \\\\\frac{-5x < -15}{-5}\\\\\boxed{x > 3}[/tex]

Karina uses the system of equations below to compare the monthly utility costs in July and December for electricity, x, and natural gas, y. 750x + 17y = 141.61 300x + 30y = 75.90 Karina solves the system using linear combination and arrives at the equation 116y = 96.28. She then solves this equation for y. Which statement explains Karina’s solution? The cost of natural gas is $0.17 per unit. The cost of natural gas is $0.20 per unit. The cost of natural gas is $0.72 per unit. The cost of natural gas is $0.83 per unit.

Answers

The correct statement is The cost of natural gas is $0.83 per unit.

The equations representing her utility costs in July and December are known as simultaneous equations. These equations have to be solved jointly in order to determine the cost of electricity and natural gas.

Given the equation gotten by Karina: 116y = 96.28

Where y represent the cost of natural gas

In order to determine the value of y, divide both sides of the equation by 116

y = 96.28 / 116

y = $0.83

To learn more about simultaneous equations, please check: https://brainly.com/question/25875552

Other Questions
Which describes body composition?a)They block pain messages from reaching brain cells.b)They lower blood cholesterol levels.c)They allow the heart to pump more blood with less effort.d)They increase the bodys basal metabolic rate. In a school election, 3/4 of the students vote. There are 1484 votes. Find the number of students.There are a total ofstudents. foods that is cooked cooled and reheated must be reheated to 300 / 2(0.5 + 4.5)^2 show your work what is the answer, i need help Convert 6432 Decimal to Binary Number system ? Kindly don't spam !! The ___________________ contains receptors for monitoring the position of the head. Food handlers are allowed to wear false fingernails as long as they have been applied by a professional. Set up an algebraic equation then solve. 1. When 3 is subtracted from the sum of a number and 10 the result is 2. The sum of 3 times a number and 12 is equal to 3. Find the number. 3. Three times the sum of a number and 6 is equal to 5 times the number. Find the number. 4. The width of a rectangle is 4 inches less than its length. If the perimeter measures 72 inches, find the dimensions of the rectangle. 5. The perimeter of a square is 48 inches. Find the length of each 2. Find the number. side. Question 9The probability that a vehicle will change lanes while making a turn is 55%.Suppose a random sample of 7 vehicles are observed making turns at abusy intersection.Calculate the expected value. If you use a rubric, you can grade your assignment before you turn it in. Please select the best answer from the choices provided T F. Use the table. What is the total amount of electricity a computer, a television, and a heater use in 1 hour? Show your work. Which Nation has 40 percent of its nation represented by christians? what is 13 divided by 4732 Identify China and Japan on the map by placing the letter that corresponds on the map to the Countries below.China Japan simplify\:\frac{\sqrt{5}}{\sqrt{3}} solve it will mark brain list Which of the following best describes how the amount of DNA in the cell changes during M phase?The amount of D N A doubles as the D N A is replicated.The amount of D N A slightly increases as a result of new organelle synthesis.The amount of D N A does not change while the cell grows.The amount of DNA is halved as the cell divides into two daughter cells. Which quantity is proportional to 40/8? Please Help Question in picture